PT83.S1.Q12 - Legislator: the recently passed

K 440244K 440244 Alum Member
edited February 2018 in Logical Reasoning 104 karma

Can someone please explain this flaw? Why would it matter if the election outcome was different if the majority party had not supported the bill?

Comments

  • 1000001910000019 Alum Member
    3279 karma

    As I read this question, the first thing that I was thinking was that we have no idea whether the other parties supported the bill. If a rival party had also supported the bill's passage, but gained the dozen seats than the reasoning is flawed. The credited answer choice addresses this. Does that make sense?

  • J.Y. PingJ.Y. Ping Administrator Instructor
    14156 karma

    @10000019 Good response!

    @"K 440244" Think about the assumption the argument is making. Premises say that (1) the majority party supported the bill which recently passed; and (2) polls predict they'll lose a dozen seats in the next election. Are we to assume that (1) caused (2)? That because of their support for the bill, they're going to get punished? You don't want to do that because there's no reason to. Given all the information we have, it's just as likely that (1) caused (2) as it is that (2) happened in spite of (1), meaning everyone actually loved that the party supported the bill and got it passed. (Finally! Our shitty highways are going to get fixed.) That's not why voters are predicted to oust a dozen of them. So, why are voters predicted to oust a dozen of them? We just don't know. It could be for any number of reasons. Maybe it's because their leader thinks that when he's in a locker room, it's funny to brag about sexually assaulting women. No. That's really important to realize.

    So, to put the assumption (what I wrote above as (1) caused (2)) the argument makes into words, it could go something like this: the predicted ousting owes to the party's support of the bill. Or in other words, if the party had not supported the bill, the predicted ousting would not be so. (A) says that we're given no reason to believe that assumption.

  • lTexlawzlTexlawz Free Trial Member
    277 karma

    I will give a shot and go why. Here is thing. We are looking for the central assumption. The assumption to stimulus is saying that if the law makers will lose the election because they supported a bill that was unpopular with the voters.

    Let go the answer choices and start eliminating wrong choices before we get to the right one

    Here is the diagram of the argument

    LE->LSB

    Could the law makers lose the election if they have not supported the bill? It is possible. We want an answer choice that matches the central assumption

    answer choice A states that there is no reason that the election would be different if the lawmakers didn't support the billl. This looks exactly what we want,so we will keep this for now.

    Answer choice B states about popularity of bill to exclusive of merit. This is out of scope. The stimulus doesn't mention popularity to merit. It is wrong

    Answer choice C is wrong because we are concern about the lawmakers losing the election because they support the bill. The answer choice make it seem like a method of reasoning question.We are not looking for the role of the argument,but we are looking for the flaw.

    Answer choice D is wrong. We don't know if a legislator wanted the bill to be unpopular for the sake making it unpopular. We are focus on the support it causing them to lose the election.

    Answer choice E is wrong. The focus is not an appeal to authority flaw. The focus the losing the election because the law makers supported the bill.

    Answer choice A fits the bill and is the right answer. I hope this helps explain it.

Sign In or Register to comment.